문제
다음 식을 만족하는 non-constant 다항식 P(x),Q(x)가 존재하는지 판별하시오.
P(x)10+P(x)9=Q(x)21+Q(x)20
스포 방지선
________________________________________________________________________________________________________________________
________________________________________________________________________________________________________________________
이 문제도 얘들이 많이 못 풀었는데, 내가 시험장에서 깔끔하게 푼 문제다. 여기서도 점수 좀 딴 듯 ㅋㅋ
(증명) 불가능하다. 귀류법으로 보이자. 조건을 만족하는 non-constant 다항식 P,Q가 존재한다 가정하자.
다항식 f에 대하여 Sf를 f(x)=0의 (복소수) 해집합이라 하자. 중근은 한 번만 센다. 다음 관찰을 하자.
관찰 1. SP∩SP+1=SQ∩SQ+1=∅이고, SP∪SP+1=SQ∪SQ+1이다.
관찰 2. 주어진 식 양변의 차수를 비교하면, deg(P)=21x,deg(Q)=10x인 자연수 x가 있음을 알 수 있다.
관찰 3. |Sf|≤deg(f)가 성립한다. 그러므로 |SP|+|SP+1|=|SQ|+|SQ+1|≤20x다.
이제 여기서 다음 Lemma를 보이자.
Lemma. non-constant 다항식 f에 대하여, |Sf|+|Sf+1|≥deg(f)+1가 성립한다.
Proof of Lemma. 우선 f(x)=n∏i=1(x−ri)ci, f(x)+1=m∏i=1(x−r′i)c′i라 하자.
여기서 n=|Sf|, m=|Sf+1|이다. ri,r′i들은 앞선 관찰 1에 의해 모두 서로 다른 복소수다.
한편, f′(x)는 ri를 ci−1-중근으로 가지고, r′i를 c′i−1-중근으로 가진다. 그러므로
deg(f)−1=deg(f′)≥n∑i=1(ci−1)+m∑i=1(c′i−1)=2deg(f)−n−m
가 성립하고, 정리하면 |Sf|+|Sf+1|≥deg(f)+1을 얻는다. ◼
Lemma와 관찰 3으로 20x≥|SQ|+|SQ+1|=|SP|+|SP+1|≥deg(P)+1=21x+1을 얻고, 이는 모순. ◼
'수학 > 경시 수학' 카테고리의 다른 글
인상 깊었던 MO 문제 #6: China TST 2018 Day 2 2번 (0) | 2019.05.04 |
---|---|
인상 깊었던 MO 문제 #5: USA Winter TST 2019 2번 (0) | 2019.05.03 |
인상 깊은 MO 문제 #4: USAMO 2019 3번 (0) | 2019.05.02 |
인상 깊었던 MO 문제 #3: RMM 2019 3번 (0) | 2019.04.29 |
인상 깊었던 MO 문제 #1: 2017-2018 겨울학교 모의고사 시험 2 문제 5번 (0) | 2018.09.30 |